Ordinary Differential Equation - Comparing 2 Solutions

Click For Summary
The discussion focuses on solving the ordinary differential equation involving functions g(s) and f(-s) with a parameter μ. One participant successfully derived the correct solution using integration by parts and an integrating factor, while another participant encountered a sign error in their alternative approach, leading to confusion. The incorrect method resulted in a final expression that differed by a sign, prompting a request for clarification on the mistake. The participants discussed the implications of swapping integration limits and the substitution of variables, emphasizing the importance of careful handling of signs in integration. Ultimately, the thread highlights the challenges of solving differential equations and the need for precision in mathematical manipulations.
AntSC
Messages
65
Reaction score
3

Homework Statement



{g}'\left ( s \right )+\mu g\left ( s \right )={f}'\left ( -s \right )+\mu f\left ( -s \right )
Integrate up to get
g\left ( s \right )=-f\left ( -s \right )+2\mu e^{-\mu s}\int_{-s}^{\infty }e^{-\mu {s}'}f\left ( {s}' \right )d{s}'

Homework Equations



As above

The Attempt at a Solution



I've seen a few attempts at this, one that arrives at the correct solution, which is the method i used and had no trouble with. However, another attempt I've seen, which doesn't yield the correct solution is causing me much headache as i can't work out where it's going wrong. Some pointing out here would be really helpful. Thanks

Using the following integrating factor on the equation above
e^{\int \mu ds}=e^{\mu s}
e^{\mu s}{g}'\left ( s \right )+e^{\mu s}\mu g\left ( s \right )=e^{\mu s}\left [ {f}'\left ( -s \right )+\mu f\left ( -s \right ) \right ]
At this point i wrote
\frac{d}{ds}\left [ e^{\mu s}g\left ( s \right ) \right ]=e^{\mu s}\left [ {f}'\left ( -s \right )+\mu f\left ( -s \right ) \right ]
\left. e^{\mu {s}'}g\left ( {s}' \right ) \right|_{-\infty }^{s}=\int_{-\infty }^{s}e^{\mu {s}'}\left [ {f}'\left ( {-s}' \right )+\mu f\left ( {-s}' \right ) \right ]d{s}'
Then to skip a few steps i did by parts integration to get the following (can include missing steps if needed)
e^{\mu s}g\left ( s \right )=-e^{\mu s}f\left ( -s \right )+ 2\mu \int_{-\infty }^{s}f\left ( {-s}' \right )e^{\mu {s}'}d{s}'
g\left ( s \right )=-f\left ( -s \right )+ 2\mu e^{-\mu s}\int_{-\infty }^{s}f\left ( {-s}' \right )e^{\mu {s}'}d{s}'
Swapping limits and changing 's' for '-s' gives
g\left ( s \right )=-f\left ( -s \right )+ 2\mu e^{-\mu s}\int_{-s}^{\infty }f\left ( {s}' \right )e^{-\mu {s}'}d{s}'
As required


The other route wrote took the following path
\frac{d}{ds}\left [ e^{\mu s}g\left ( s \right ) \right ]e^{-\mu s}=\frac{d}{ds}\left [ f\left ( -s \right )e^{-\mu s} \right ]e^{\mu s}
\frac{d}{ds}\left [ e^{\mu s}g\left ( s \right ) \right ]=-\frac{d}{ds}\left [ f\left ( -s \right )e^{-\mu s} \right ]e^{2\mu s}
\left. e^{\mu {s}'}g\left ( {s}' \right ) \right|_{-\infty }^{s}=\int_{-\infty }^{s}-e^{2\mu {s}'}\frac{d}{ds}\left [ f\left ( {-s}' \right )e^{-\mu {s}'} \right ]d{s}'
e^{\mu s}g\left ( s \right )=\left. -e^{2\mu {s}'}f\left ( {-s}' \right )e^{-\mu {s}'}\right|_{-\infty }^{s}+ \int_{-\infty }^{s}2\mu e^{2\mu s}f\left ( {-s}' \right )e^{-\mu {s}'}d{s}'
e^{\mu s}g\left ( s \right )=-e^{\mu s}f\left ( -s \right )+ 2\mu e^{2\mu s}\int_{-\infty }^{s}f\left ( {-s}' \right )e^{-\mu {s}'}d{s}'
g\left ( s \right )=-f\left ( -s \right )+ 2\mu e^{\mu s}\int_{-\infty }^{s}f\left ( {-s}' \right )e^{-\mu {s}'}d{s}'
Swapping limits and changing 's' for '-s' gives
g\left ( s \right )=-f\left ( -s \right )+ 2\mu e^{\mu s}\int_{-s}^{\infty }f\left ( {s}' \right )e^{\mu {s}'}d{s}'
This is giving a sign swap in the final solutioin that is having my head spin. Can't see the route of this. Can anyone spot the mistake?
 
Physics news on Phys.org
Swapping \int_{-s}^\infty ds' for \int_{-\infty}^s ds' is equivalent to making the substitution u= -s'. Then du= -ds'. When s'= -\infty, u= \infty and when s'= s, u= -s.

So \int_{-s}^\infty f(-s')e^{-\mu s'}ds'= \int_u^{-\infty}f(u)e^{\mu u}(-du)= \int_{-\infty}^u f(u)e^{\mu u}du
 
Question: A clock's minute hand has length 4 and its hour hand has length 3. What is the distance between the tips at the moment when it is increasing most rapidly?(Putnam Exam Question) Answer: Making assumption that both the hands moves at constant angular velocities, the answer is ## \sqrt{7} .## But don't you think this assumption is somewhat doubtful and wrong?

Similar threads

Replies
6
Views
2K
Replies
9
Views
2K
Replies
2
Views
2K
Replies
5
Views
2K
  • · Replies 6 ·
Replies
6
Views
1K
  • · Replies 15 ·
Replies
15
Views
2K
Replies
3
Views
1K
  • · Replies 3 ·
Replies
3
Views
1K
  • · Replies 12 ·
Replies
12
Views
1K
Replies
2
Views
1K